Đến nội dung

Hình ảnh

Topic tổng hợp một số bất đẳng thức trong kì thi MO các nước

* * * * * 16 Bình chọn

  • Please log in to reply
Chủ đề này có 501 trả lời

#101
Hoang Tung 126

Hoang Tung 126

    Thiếu tá

  • Thành viên
  • 2061 Bài viết

Bài 43 : Cho $x,y,z$ là các số thực dương . CMR : 

$\sum \frac{(y+z)(x+z)}{(x+y)(x+y+2z)}\geq \frac{3}{2}$

Đặt $y+z=a,x+z=b,x+y=c= > \sum \frac{(y+z)(x+z)}{(x+y)(x+y+2z)}=\sum \frac{ab}{c(a+b)}=\sum \frac{(ab)^2}{a^2bc+ab^2c}\geq \frac{(\sum ab)^2}{2abc\sum a}\geq \frac{3abc\sum a}{2abc\sum a}=\frac{3}{2}$



#102
Bui Ba Anh

Bui Ba Anh

    Thiếu úy

  • Thành viên
  • 562 Bài viết

Bài 44: 

Áp dụng BĐT BCS ta có:

$(a+b+c+d)^2\leq (1+2+3+4)(a^2+\frac{b^2}{2}+\frac{c^2}{3}+\frac{d^2}{4}=\frac{5(12a^2+6b^2+4c^2+3d^2}{6}$

Chú ý rằng $12a^2+6b^2+4c^2+3d^2=6a^2+2(a^2+b^2)+(a^2+b^2+c^2)+3(a^2+b^2+c^2+d^2)\leq 120$

Nên $(a+b+c+d)^2\leq 100=>a+b+c+d\leq |a+b+c+d|\leq 10$

Dấu bằng xảy ra khi $(a,b,c,d)=(1,2,3,4)$


NgọaLong

#103
longatk08

longatk08

    Sĩ quan

  • Thành viên
  • 350 Bài viết

 

Bài 41:

Cho $x,y,z$ là các số dương mà $xy+yz+zx+xyz=4$.CMR

$x^2+y^2+z^2+3+6\sqrt{2}\geq (2+2\sqrt{2})(x+y+z)$

Điều kiện bài toán chỉ phụ thuộc vào $xy+yz+xz,xyz$ nên ta có thể cố định 2 đại lượng này và thay đổi $x+y+z$.

 

Khi đó đặt $p=x+y+z,q=xy+yz+xz,r=xyz$.BĐT tương đương với:

 

$f(p)=p^2-(2+2\sqrt{2})p-2q+3+6\sqrt{2}$

 

Từ điều kiện nài toán ta có thể suy ra $p\geq 3$.Vậy nên hàm theo $p$ đồng biến.Do đó hàm này sẽ đạt giá trị nhỏ nhất khi $a+b+c$ nhỏ nhất.

Theo định lý ABC mở rộng thì $a+b+c$ nhỏ nhất khi có 2 trong 3 số bằng nhau ta giả sử là $x=y$. Khi đó ta thế $z$ theo $x$ xuống thì được cái này đây.

 

Cái đồ thị hơi dị


Bài viết đã được chỉnh sửa nội dung bởi longatk08: 22-05-2015 - 18:41


#104
khanghaxuan

khanghaxuan

    Trung úy

  • Thành viên
  • 969 Bài viết

Bài 43 :  Với cách đặt : $\left\{\begin{matrix} a=\frac{x+y}{y+z} & & \\ b=\frac{y+z}{z+x} & & \\ c=\frac{z+x}{x+y} & & \end{matrix}\right.$

Ta có bài toán quen thuộc sau : 

$\frac{1}{a(1+b)}+\frac{1}{b(1+c)}+\frac{1}{c(1+a)}\geq \frac{3}{1+abc}$ :)


Điều tôi muốn biết trước tiên không phải là bạn đã thất bại ra sao mà là bạn đã chấp nhận nó như thế nào .

- A.Lincoln -

#105
Bui Ba Anh

Bui Ba Anh

    Thiếu úy

  • Thành viên
  • 562 Bài viết

Bài 43 :  Với cách đặt : $\left\{\begin{matrix} a=\frac{x+y}{y+z} & & \\ b=\frac{y+z}{z+x} & & \\ c=\frac{z+x}{x+y} & & \end{matrix}\right.$

Ta có bài toán quen thuộc sau : 

$\frac{1}{a(1+b)}+\frac{1}{b(1+c)}+\frac{1}{c(1+a)}\geq \frac{3}{1+abc}$ :)

Có thể chứng minh ngược lại là: với mọi $a,b,c$ luôn tồn tại cách đặt như trên không nhỉ?

Nếu được thì hẳn là một đổi biến mới 


NgọaLong

#106
nhungvienkimcuong

nhungvienkimcuong

    Thiếu úy

  • Hiệp sỹ
  • 678 Bài viết

Theo định lý ABC mở rộng thì $a+b+c$ nhỏ nhất khi có 2 trong 3 số bằng nhau ta giả sử là $x=y$. 

định lí $ABC$ là một định lí rất mạnh mà do anh Nguyễn Anh Cường đưa ra nhưng trong các cuộc thi khi dùng cái này bị trừ điểm không ít còn nếu không muốn trừ điểm thì phải chứng minh lại nhiều thứ phức tạp,mất thời gian nên theo em thì các bài toán ở topic đừng nên lạm dụng quá một cái "dao không được dùng" như thế này

mong anh và mọi người sẽ có lời giải khác cho bài $41$


Bài viết đã được chỉnh sửa nội dung bởi nhungvienkimcuong: 22-05-2015 - 18:17

Đừng khóc vì chuyện đã kết thúc hãy cười vì chuyện đã xảy ra ~O) 
Thật kì lạ anh không thể nhớ đến tên mình mà chỉ nhớ đến tên em :wub:
Chúa tạo ra vũ trụ của con người còn em tạo ra vũ trụ của anh :ukliam2:


#107
longatk08

longatk08

    Sĩ quan

  • Thành viên
  • 350 Bài viết

định lí $ABC$ là một định lí rất mạnh mà do anh Nguyễn Anh Cường đưa ra nhưng trong các cuộc thi khi dùng cái này bị trừ điểm không ít còn nếu không muốn trừ điểm thì phải chứng minh lại nhiều thứ phức tạp,mất thời gian nên theo em thì các bài toán ở topic đừng nên lạm dụng quá một cái "dao không được dùng" như thế này

mong anh và mọi người sẽ có lời giải khác cho bài $41$

Vậy để anh thử p,q,r vậy đằng nào cũng có hàm đồng biến rồi mà :D



#108
dogsteven

dogsteven

    Đại úy

  • Thành viên
  • 1567 Bài viết

Bài 41. 

Giả sử $x=\text{min}\{x,y,z\}$, Thay trực tiếp $x=\dfrac{4-yz}{y+z+yz}$ ta được:

$VT-VP=y^2+z^2+\dfrac{(4-yz)^2}{y+z+yz}-2\left(1+\sqrt{2}\right)\left(y+z+\dfrac{4-yz}{y+z+yz}\right)+3+6\sqrt{2}$

Đặt $a=y+z$ và $b=yz$ thì ta có $f(b)=a^2-2b+\dfrac{(4-b)^2}{(a+b)^2}-2(1+\sqrt{2})\left(a+\dfrac{4-b}{a+b}\right)+3+6\sqrt{2}$

$=-2b+\dfrac{4-b}{(a+b)^2}\left[4-b-2(1+\sqrt{2})(a+b)\right]+a^2-2(1+\sqrt{2})a+3+6\sqrt{2}$

Có:

$4-b-2(1+\sqrt{2})(a+b)\geqslant 4-\dfrac{a^2}{4}-2(1+\sqrt{2})\left(a+\dfrac{a^2}{4}\right)$

$4-b\geqslant 4-\dfrac{a^2}{4}$

$-2b\geqslant \dfrac{a^2}{2}$

$\dfrac{1}{(a+b)^2}\geqslant \dfrac{1}{\left(a+\dfrac{a^2}{4}\right)^2}$

Do đó ta chỉ cần chứng minh khi $b=\dfrac{a^2}{4}$


Bài viết đã được chỉnh sửa nội dung bởi dogsteven: 22-05-2015 - 20:09

Quyết tâm off dài dài cày hình, số, tổ, rời rạc.


#109
khanghaxuan

khanghaxuan

    Trung úy

  • Thành viên
  • 969 Bài viết

Bài 45 : Cho $x,y,z$ thỏa mãn : $x+y+z=0$ . Chứng minh rằng : 

$\frac{x(x+2)}{2x^{2}+1}+\frac{y(y+2)}{2y^{2}+1}+\frac{z(z+2)}{2z^{2}+1}\geq 0$ :))


Điều tôi muốn biết trước tiên không phải là bạn đã thất bại ra sao mà là bạn đã chấp nhận nó như thế nào .

- A.Lincoln -

#110
tonarinototoro

tonarinototoro

    Trung sĩ

  • Thành viên
  • 174 Bài viết

Bài 45 : Cho $x,y,z$ thỏa mãn : $x+y+z=0$ . Chứng minh rằng : 

$\frac{x(x+2)}{2x^{2}+1}+\frac{y(y+2)}{2y^{2}+1}+\frac{z(z+2)}{2z^{2}+1}\geq 0$ :))

$BĐT\Leftrightarrow \frac{x\left ( x+2 \right )}{2x^{2}+1}+\frac{1}{2}+\frac{y\left ( y+2 \right )}{2y^{2}+1}+\frac{1}{2}\geq 1-\frac{z\left ( z+2 \right )}{2z^{2}+1}\Leftrightarrow \frac{\left ( 2x+1 \right )^{2}}{2x^{2}+1}+\frac{\left ( 2y+1 \right )^{2}}{2y^{2}+1}\geq \frac{2\left ( z-1 \right )^{2}}{2z^{2}+1}$

Cauchy-Schwarz :$\frac{\left ( 2x+1 \right )^{2}}{2x^{2}+1}+\frac{\left ( 2y+1 \right )^{2}}{2y^{2}+1}\geq \frac{2\left ( x+y+1 \right )^{2}}{x^{2}+y^{2}+1}= \frac{\left ( z-1 \right )^{2}}{2z^{2}+1}$

Ta chỉ cần chứng minh $x^{2}+y^{2}+1\leq 2z^{2}+1\rightarrow $ điều này đúng nếu giả sử $\left | z \right |=max\left \{ \left | x \right |;\left | y \right | ;\left | z \right |\right \}$

Vậy bài toán được chứng minh. Đẳng thức xảy ra khi $x=y=z=0$



#111
khanghaxuan

khanghaxuan

    Trung úy

  • Thành viên
  • 969 Bài viết

Bài 46 : Cho $a,b,c>0$ thỏa mãn : $a+b+c=1$ . CMR : 

$\sum \frac{a}{b}+\frac{b}{a}+6\geq 2\sqrt{2}(\sum \sqrt{\frac{1-a}{a}})$

 

Bài 47 : Cho $a,b,c>0$ thỏa mãn : $abc=1$ . CMR : 

$\frac{1}{a^{3}+bc}+\frac{1}{b^{3}+ca}+\frac{1}{c^{3}+ab}\leq \frac{(ab+bc+ca)^{2}}{6}$

 

Bài 48 : Cho $a,b,c>0$ thỏa mãn : $a+b+c=a^{2}+b^{2}+c^{2}$ . CMR : 

$\frac{a^{2}}{a^{2}+ab}+\frac{b^{2}}{b^{2}+bc}+\frac{c^{2}}{c^{2}+ca}\geq \frac{a+b+c}{2}$

 

P/s : Dễ xơi :)


Bài viết đã được chỉnh sửa nội dung bởi khanghaxuan: 23-05-2015 - 19:58

Điều tôi muốn biết trước tiên không phải là bạn đã thất bại ra sao mà là bạn đã chấp nhận nó như thế nào .

- A.Lincoln -

#112
khanghaxuan

khanghaxuan

    Trung úy

  • Thành viên
  • 969 Bài viết

Bài 49 : Cho $a,b,c>0$ thỏa mãn : $abc=1$ . CMR : 

$\frac{a^{4}}{3a+3ab+2}+\frac{b^{4}}{3b+3bc+2}+\frac{c^{4}}{3c+3ca+2}\geq \frac{3}{8}$ :)


Bài viết đã được chỉnh sửa nội dung bởi khanghaxuan: 24-05-2015 - 08:04

Điều tôi muốn biết trước tiên không phải là bạn đã thất bại ra sao mà là bạn đã chấp nhận nó như thế nào .

- A.Lincoln -

#113
ducvipdh12

ducvipdh12

    Sĩ quan

  • Thành viên
  • 454 Bài viết

mình vừa gửi 1 file bữa trước cho anh Cẩn xem,anh khen hay nhưng anh có khuyên 1 cái là khi giải thì các bạn hãy phân tích lời giải,hướng giải của mình,như thế mới có thể khác với các sách BĐT khác được :))


Bài viết đã được chỉnh sửa nội dung bởi ducvipdh12: 23-05-2015 - 20:45

FAN THẦY THÔNG,ANH CẨN,THẦY VINH :icon6: :icon6:

#114
nhungvienkimcuong

nhungvienkimcuong

    Thiếu úy

  • Hiệp sỹ
  • 678 Bài viết

mình vừa gửi 1 file bữa trước cho anh Cẩn xem,anh khen hay nhưng anh có khuyên 1 cái là khi giải thì các bạn hãy phân tích lời giải,hướng giải của mình,như thế mới có thể khác với các sách BĐT khác được :))

ok luôn anh,mà em Văn hơi dốt

p/s:fan anh Cẩn điểm danh


Đừng khóc vì chuyện đã kết thúc hãy cười vì chuyện đã xảy ra ~O) 
Thật kì lạ anh không thể nhớ đến tên mình mà chỉ nhớ đến tên em :wub:
Chúa tạo ra vũ trụ của con người còn em tạo ra vũ trụ của anh :ukliam2:


#115
Bui Ba Anh

Bui Ba Anh

    Thiếu úy

  • Thành viên
  • 562 Bài viết

Bài 49 : Cho $a,b,c>0$ . CMR : 

$\frac{a^{4}}{3a+3ab+2}+\frac{b^{4}}{3b+3bc+2}+\frac{c^{4}}{3c+3ca+2}\geq \frac{3}{8}$ :)

BĐT k đồng bậc sao k có thêm điều kiện nhỉ


NgọaLong

#116
binhnhaukhong

binhnhaukhong

    Sĩ quan

  • Thành viên
  • 343 Bài viết

Bài 46 : Cho $a,b,c>0$ thỏa mãn : $a+b+c=1$ . CMR : 

$\sum \frac{a}{b}+\frac{b}{a}+6\geq 2\sqrt{2}(\sum \sqrt{\frac{1-a}{a}})$

 

Bài 47 : Cho $a,b,c>0$ thỏa mãn : $abc=1$ . CMR : 

$\frac{1}{a^{3}+bc}+\frac{1}{b^{3}+ca}+\frac{1}{c^{3}+ab}\leq \frac{(ab+bc+ca)^{2}}{6}$

 

Bài 48 : Cho $a,b,c>0$ thỏa mãn : $a+b+c=a^{2}+b^{2}+c^{2}$ . CMR : 

$\frac{a^{2}}{a^{2}+ab}+\frac{b^{2}}{b^{2}+bc}+\frac{c^{2}}{c^{2}+ca}\geq \frac{a+b+c}{2}$

 

P/s : Dễ xơi :)

Ôi những bài BĐT cổ điển :D

 

1.

BĐT tương đương với:

$\frac{a+b}{c}+\frac{b+c}{a}+\frac{a+c}{b}+6\geq 2\sqrt{2}(\sum \sqrt{\frac{a+b}{c}})$

 

BĐT trên là tổng của các BĐT 

$\frac{a+b}{c}+2\geq 2\sqrt{2}\sqrt{\frac{a+b}{c}}$ và tương tự

 

2.Dễ thấy BĐT trên tương đương với:

$\frac{a}{a^4+1}+\frac{b}{b^4+1}+\frac{c}{c^4+1}\leq \frac{(ab+bc+ac)^2}{6}$

 

Áp dụng BĐT AM-GM ta có $a^4+1\geq 2a^2$

Do đó ta quy về chứng minh:

$\frac{1}{a}+\frac{1}{b}+\frac{1}{c}\leq \frac{(ab+bc+ac)^2}{3}$

$\Leftrightarrow \frac{ab+bc+ac}{3}\geq 1$

 

BĐT trên đúng với giả thiết $abc=1$

 

3.Áp dụng BĐT C-S và giả thiết thì BĐT của ta tương đương với:

$a^2+b^2+c^2\geq ab+bc+ac$


Quy Ẩn Giang Hồ. 

So goodbye!

 

:off:  :off:  :off:  :off:  :off:  :off: 


#117
tonarinototoro

tonarinototoro

    Trung sĩ

  • Thành viên
  • 174 Bài viết

Bài 49 : Cho $a,b,c>0$ . CMR : 

$\frac{a^{4}}{3a+3ab+2}+\frac{b^{4}}{3b+3bc+2}+\frac{c^{4}}{3c+3ca+2}\geq \frac{3}{8}$ :)

đề bài có thiếu không nhỉ. $a=b=c=0,5$ thì bđt sai     :closedeyes:  



#118
nguyenhongsonk612

nguyenhongsonk612

    Thượng úy

  • Thành viên
  • 1451 Bài viết

 

Bài 48 : Cho $a,b,c>0$ thỏa mãn : $a+b+c=a^{2}+b^{2}+c^{2}$ . CMR : 

$\frac{a^{2}}{a^{2}+ab}+\frac{b^{2}}{b^{2}+bc}+\frac{c^{2}}{c^{2}+ca}\geq \frac{a+b+c}{2}$

 

P/s : Dễ xơi :)

Từ GT $\Rightarrow \frac{a+b+c}{a^2+b^2+c^2}=1$

BĐT cần C/m có thể được viết lại dưới dạng 

                                 $\frac{a^2}{a^2+ab}+\frac{b^2}{b^2+bc}+\frac{c^2}{c^2+ca}\geq \frac{(a+b+c)^2}{2(a^2+b^2+c^2)}$

 Áp dụng BĐT Cauchy Schwarz ta có

                                $VT\geq \frac{(a+b+c)^2}{a^2+b^2+c^2+ab+bc+ca}\geq \frac{(a+b+c)^2}{2(a^2+b^2+c^2)}$

Vậy ta có đpcm

Dấu "=" xảy ra khi và chỉ khi $a=b=c=1$


"...Từ ngay ngày hôm nay tôi sẽ chăm chỉ học hành như Stardi, với đôi tay nắm chặt và hàm răng nghiến lại đầy quyết tâm. Tôi sẽ nỗ lực với toàn bộ trái tim và sức mạnh để hạ gục cơn buồn ngủ vào mỗi tối và thức dậy sớm vào mỗi sáng. Tôi sẽ vắt óc ra mà học và không nhân nhượng với sự lười biếng. Tôi có thể học đến phát bệnh miễn là thoát khỏi cuộc sống nhàm chán khiến mọi người và cả chính tôi mệt mỏi như thế này. Dũng cảm lên! Hãy bắt tay vào công việc với tất cả trái tim và khối óc. Làm việc để lấy lại niềm vui, lấy lại nụ cười trên môi thầy giáo và cái hôn chúc phúc của bố tôi. " (Trích "Những tấm lòng cao cả")

~O) 


#119
khanghaxuan

khanghaxuan

    Trung úy

  • Thành viên
  • 969 Bài viết

Bài 49 đã fix (Quên cái Đk :))

 

P/s : Theo mình thì từ bài 50 trở đi thì các bạn nên phân tích bài toán một chút nhé ! Thân :)


Bài viết đã được chỉnh sửa nội dung bởi khanghaxuan: 24-05-2015 - 08:12

Điều tôi muốn biết trước tiên không phải là bạn đã thất bại ra sao mà là bạn đã chấp nhận nó như thế nào .

- A.Lincoln -

#120
khanghaxuan

khanghaxuan

    Trung úy

  • Thành viên
  • 969 Bài viết

Lời giải bài 49 :

Bổ đề :  $a^{2}+b^{2}+c^{2}+2abc+1\geq 2(ab+bc+ca)$

              $a^{2}+b^{2}+c^{2}+3\geq 2(a+b+c)$

$\Rightarrow 2(\sum a^{2})+2abc+4\geq 2\sum a+2\sum ab$

$\Rightarrow (1+a)(1+b)(1+c)\leq \sum a^{2}+5$

Mặt khác , áp dụng C-S ta được : 

$\sum \frac{a^{4}}{a+ab+\frac{2}{3}}\geq \frac{(\sum a^{2})^{2}}{\sum a+\sum ab+2}=\frac{(\sum a^{2})^{2}}{\prod (1+a)}\geq \frac{(\sum a^{2})^{2}}{\sum a^{2}+5}$

Ta cần chứng minh : $\frac{(\sum a^{2})^{2}}{\sum a^{2}+5}\geq \frac{9}{8}\Leftrightarrow \sum a^{2}\geq 3$ (luôn đúng :))

Vậy BĐT được chứng minh trọn vẹn :))


Điều tôi muốn biết trước tiên không phải là bạn đã thất bại ra sao mà là bạn đã chấp nhận nó như thế nào .

- A.Lincoln -




1 người đang xem chủ đề

0 thành viên, 1 khách, 0 thành viên ẩn danh